Sr Examen

Otras calculadoras

log^3(2x+1)<3 desigualdades

En la desigualdad la incógnita

Solución

Ha introducido [src]
   3             
log (2*x + 1) < 3
$$\log{\left(2 x + 1 \right)}^{3} < 3$$
log(2*x + 1)^3 < 3
Solución detallada
Se da la desigualdad:
$$\log{\left(2 x + 1 \right)}^{3} < 3$$
Para resolver esta desigualdad primero hay que resolver la ecuación correspondiente:
$$\log{\left(2 x + 1 \right)}^{3} = 3$$
Resolvemos:
$$x_{1} = - \frac{1}{2} + \frac{e^{\sqrt[3]{3}}}{2}$$
$$x_{2} = - \frac{1}{2} + \frac{e^{- \frac{\sqrt[3]{3}}{2} - \frac{3^{\frac{5}{6}} i}{2}}}{2}$$
$$x_{3} = - \frac{1}{2} + \frac{e^{- \frac{\sqrt[3]{3}}{2} + \frac{3^{\frac{5}{6}} i}{2}}}{2}$$
Descartamos las soluciones complejas:
$$x_{1} = - \frac{1}{2} + \frac{e^{\sqrt[3]{3}}}{2}$$
Las raíces dadas
$$x_{1} = - \frac{1}{2} + \frac{e^{\sqrt[3]{3}}}{2}$$
son puntos de cambio del signo de desigualdad en las soluciones.
Primero definámonos con el signo hasta el punto extremo izquierdo:
$$x_{0} < x_{1}$$
Consideremos, por ejemplo, el punto
$$x_{0} = x_{1} - \frac{1}{10}$$
=
$$- \frac{1}{10} + \left(- \frac{1}{2} + \frac{e^{\sqrt[3]{3}}}{2}\right)$$
=
$$- \frac{3}{5} + \frac{e^{\sqrt[3]{3}}}{2}$$
lo sustituimos en la expresión
$$\log{\left(2 x + 1 \right)}^{3} < 3$$
$$\log{\left(1 + 2 \left(- \frac{3}{5} + \frac{e^{\sqrt[3]{3}}}{2}\right) \right)}^{3} < 3$$
    /       3 ___\    
   3|  1    \/ 3 |    
log |- - + e     | < 3
    \  5         /    
    

significa que la solución de la desigualdad será con:
$$x < - \frac{1}{2} + \frac{e^{\sqrt[3]{3}}}{2}$$
 _____          
      \    
-------ο-------
       x1
Solución de la desigualdad en el gráfico
Respuesta rápida 2 [src]
              3 ___ 
              \/ 3  
         1   e      
(-1/2, - - + ------)
         2     2    
$$x\ in\ \left(- \frac{1}{2}, - \frac{1}{2} + \frac{e^{\sqrt[3]{3}}}{2}\right)$$
x in Interval.open(-1/2, -1/2 + exp(3^(1/3))/2)
Respuesta rápida [src]
   /                     3 ___\
   |                     \/ 3 |
   |                1   e     |
And|-1/2 < x, x < - - + ------|
   \                2     2   /
$$- \frac{1}{2} < x \wedge x < - \frac{1}{2} + \frac{e^{\sqrt[3]{3}}}{2}$$
(-1/2 < x)∧(x < -1/2 + exp(3^(1/3))/2)